Derangements $p$ of $1,2,dots,n,n+1$ such that $n+1$ doesn't go to $n$












4












$begingroup$


Recall that the number or Derangements of $1,2,dots,n$ is a permutation $p$ such that $p(i) neq i$ for all $i$. We can express it with the recurrence $D_n=(n-1)(D_{n-1}+D_{n-2})$ or by the closed formula $$D_n =sum_{i=0}^n (-1)^i frac{n!}{i!}$$



Now we consider the number of permutations of $1,2,dots, n,n+1$ such that for all $1leq i leq n$ (not $n+1$), $p(i)neq i$, but also $p(n+1) neq n$. We need to express this number using $D_n$s.



I was thinking of first counting the number of permutations without the additional condition $p(n+1) neq n$ and received using inclusion-exclusion



$$sum_{r=0}^n (-1)^r binom{n}{r}(n+1-r)! = sum_{r=0}^n (-1)^r frac{n!}{r!}(n+1-r) =$$
$$sum_{r=0}^n (-1)^r frac{(n+1)!}{r!}- sum_{r=1}^n (-1)^r frac{n!}{(r-1)!}=$$
$$sum_{r=0}^n (-1)^r frac{(n+1)!}{r!}- nsum_{r=0}^{n-1} (-1)^{r+1} frac{(n-1)!}{r!}=$$
$$=D_{n+1}+nD_{n-1}$$



Now we need to subtract from this the number of permutations when $p(n+1)=n$, which I wasn't able to count.



Thanks in advance










share|cite|improve this question











$endgroup$












  • $begingroup$
    The number of permutations of ${1,ldots,n+1}$ with $p(n+1)=n$ is simply the number of permutations of ${1,ldots,n}$; compose any such permutation with the transposition $(n n+1)$ yields a bijection between these sets of permutations.
    $endgroup$
    – Servaes
    Dec 15 '18 at 9:42












  • $begingroup$
    But in our case, we count the permutations with $p(i)neq i$, are you sure there is such bijection? Because when $n+1$ takes $n$'s place, we dont have to worry about $p(n) neq n$ anymore.
    $endgroup$
    – Theorem
    Dec 15 '18 at 9:46










  • $begingroup$
    Ah, so you mean to count the number of derangements with $p(n+1)neq n$? I only read your question at a glance, and commented on the last sentence.
    $endgroup$
    – Servaes
    Dec 15 '18 at 9:49










  • $begingroup$
    Yes, I didn't say exactly derangements because that would imply $p(n+1)neq n+1$, but it is very similar.
    $endgroup$
    – Theorem
    Dec 15 '18 at 9:50










  • $begingroup$
    In a derangement, $n+1$ is as likely to go to $n$ as to any given element of ${1,ldots,n}$.
    $endgroup$
    – Lord Shark the Unknown
    Dec 15 '18 at 10:30
















4












$begingroup$


Recall that the number or Derangements of $1,2,dots,n$ is a permutation $p$ such that $p(i) neq i$ for all $i$. We can express it with the recurrence $D_n=(n-1)(D_{n-1}+D_{n-2})$ or by the closed formula $$D_n =sum_{i=0}^n (-1)^i frac{n!}{i!}$$



Now we consider the number of permutations of $1,2,dots, n,n+1$ such that for all $1leq i leq n$ (not $n+1$), $p(i)neq i$, but also $p(n+1) neq n$. We need to express this number using $D_n$s.



I was thinking of first counting the number of permutations without the additional condition $p(n+1) neq n$ and received using inclusion-exclusion



$$sum_{r=0}^n (-1)^r binom{n}{r}(n+1-r)! = sum_{r=0}^n (-1)^r frac{n!}{r!}(n+1-r) =$$
$$sum_{r=0}^n (-1)^r frac{(n+1)!}{r!}- sum_{r=1}^n (-1)^r frac{n!}{(r-1)!}=$$
$$sum_{r=0}^n (-1)^r frac{(n+1)!}{r!}- nsum_{r=0}^{n-1} (-1)^{r+1} frac{(n-1)!}{r!}=$$
$$=D_{n+1}+nD_{n-1}$$



Now we need to subtract from this the number of permutations when $p(n+1)=n$, which I wasn't able to count.



Thanks in advance










share|cite|improve this question











$endgroup$












  • $begingroup$
    The number of permutations of ${1,ldots,n+1}$ with $p(n+1)=n$ is simply the number of permutations of ${1,ldots,n}$; compose any such permutation with the transposition $(n n+1)$ yields a bijection between these sets of permutations.
    $endgroup$
    – Servaes
    Dec 15 '18 at 9:42












  • $begingroup$
    But in our case, we count the permutations with $p(i)neq i$, are you sure there is such bijection? Because when $n+1$ takes $n$'s place, we dont have to worry about $p(n) neq n$ anymore.
    $endgroup$
    – Theorem
    Dec 15 '18 at 9:46










  • $begingroup$
    Ah, so you mean to count the number of derangements with $p(n+1)neq n$? I only read your question at a glance, and commented on the last sentence.
    $endgroup$
    – Servaes
    Dec 15 '18 at 9:49










  • $begingroup$
    Yes, I didn't say exactly derangements because that would imply $p(n+1)neq n+1$, but it is very similar.
    $endgroup$
    – Theorem
    Dec 15 '18 at 9:50










  • $begingroup$
    In a derangement, $n+1$ is as likely to go to $n$ as to any given element of ${1,ldots,n}$.
    $endgroup$
    – Lord Shark the Unknown
    Dec 15 '18 at 10:30














4












4








4


2



$begingroup$


Recall that the number or Derangements of $1,2,dots,n$ is a permutation $p$ such that $p(i) neq i$ for all $i$. We can express it with the recurrence $D_n=(n-1)(D_{n-1}+D_{n-2})$ or by the closed formula $$D_n =sum_{i=0}^n (-1)^i frac{n!}{i!}$$



Now we consider the number of permutations of $1,2,dots, n,n+1$ such that for all $1leq i leq n$ (not $n+1$), $p(i)neq i$, but also $p(n+1) neq n$. We need to express this number using $D_n$s.



I was thinking of first counting the number of permutations without the additional condition $p(n+1) neq n$ and received using inclusion-exclusion



$$sum_{r=0}^n (-1)^r binom{n}{r}(n+1-r)! = sum_{r=0}^n (-1)^r frac{n!}{r!}(n+1-r) =$$
$$sum_{r=0}^n (-1)^r frac{(n+1)!}{r!}- sum_{r=1}^n (-1)^r frac{n!}{(r-1)!}=$$
$$sum_{r=0}^n (-1)^r frac{(n+1)!}{r!}- nsum_{r=0}^{n-1} (-1)^{r+1} frac{(n-1)!}{r!}=$$
$$=D_{n+1}+nD_{n-1}$$



Now we need to subtract from this the number of permutations when $p(n+1)=n$, which I wasn't able to count.



Thanks in advance










share|cite|improve this question











$endgroup$




Recall that the number or Derangements of $1,2,dots,n$ is a permutation $p$ such that $p(i) neq i$ for all $i$. We can express it with the recurrence $D_n=(n-1)(D_{n-1}+D_{n-2})$ or by the closed formula $$D_n =sum_{i=0}^n (-1)^i frac{n!}{i!}$$



Now we consider the number of permutations of $1,2,dots, n,n+1$ such that for all $1leq i leq n$ (not $n+1$), $p(i)neq i$, but also $p(n+1) neq n$. We need to express this number using $D_n$s.



I was thinking of first counting the number of permutations without the additional condition $p(n+1) neq n$ and received using inclusion-exclusion



$$sum_{r=0}^n (-1)^r binom{n}{r}(n+1-r)! = sum_{r=0}^n (-1)^r frac{n!}{r!}(n+1-r) =$$
$$sum_{r=0}^n (-1)^r frac{(n+1)!}{r!}- sum_{r=1}^n (-1)^r frac{n!}{(r-1)!}=$$
$$sum_{r=0}^n (-1)^r frac{(n+1)!}{r!}- nsum_{r=0}^{n-1} (-1)^{r+1} frac{(n-1)!}{r!}=$$
$$=D_{n+1}+nD_{n-1}$$



Now we need to subtract from this the number of permutations when $p(n+1)=n$, which I wasn't able to count.



Thanks in advance







combinatorics permutations inclusion-exclusion






share|cite|improve this question















share|cite|improve this question













share|cite|improve this question




share|cite|improve this question








edited Dec 15 '18 at 12:49







Theorem

















asked Dec 15 '18 at 9:38









TheoremTheorem

1,068513




1,068513












  • $begingroup$
    The number of permutations of ${1,ldots,n+1}$ with $p(n+1)=n$ is simply the number of permutations of ${1,ldots,n}$; compose any such permutation with the transposition $(n n+1)$ yields a bijection between these sets of permutations.
    $endgroup$
    – Servaes
    Dec 15 '18 at 9:42












  • $begingroup$
    But in our case, we count the permutations with $p(i)neq i$, are you sure there is such bijection? Because when $n+1$ takes $n$'s place, we dont have to worry about $p(n) neq n$ anymore.
    $endgroup$
    – Theorem
    Dec 15 '18 at 9:46










  • $begingroup$
    Ah, so you mean to count the number of derangements with $p(n+1)neq n$? I only read your question at a glance, and commented on the last sentence.
    $endgroup$
    – Servaes
    Dec 15 '18 at 9:49










  • $begingroup$
    Yes, I didn't say exactly derangements because that would imply $p(n+1)neq n+1$, but it is very similar.
    $endgroup$
    – Theorem
    Dec 15 '18 at 9:50










  • $begingroup$
    In a derangement, $n+1$ is as likely to go to $n$ as to any given element of ${1,ldots,n}$.
    $endgroup$
    – Lord Shark the Unknown
    Dec 15 '18 at 10:30


















  • $begingroup$
    The number of permutations of ${1,ldots,n+1}$ with $p(n+1)=n$ is simply the number of permutations of ${1,ldots,n}$; compose any such permutation with the transposition $(n n+1)$ yields a bijection between these sets of permutations.
    $endgroup$
    – Servaes
    Dec 15 '18 at 9:42












  • $begingroup$
    But in our case, we count the permutations with $p(i)neq i$, are you sure there is such bijection? Because when $n+1$ takes $n$'s place, we dont have to worry about $p(n) neq n$ anymore.
    $endgroup$
    – Theorem
    Dec 15 '18 at 9:46










  • $begingroup$
    Ah, so you mean to count the number of derangements with $p(n+1)neq n$? I only read your question at a glance, and commented on the last sentence.
    $endgroup$
    – Servaes
    Dec 15 '18 at 9:49










  • $begingroup$
    Yes, I didn't say exactly derangements because that would imply $p(n+1)neq n+1$, but it is very similar.
    $endgroup$
    – Theorem
    Dec 15 '18 at 9:50










  • $begingroup$
    In a derangement, $n+1$ is as likely to go to $n$ as to any given element of ${1,ldots,n}$.
    $endgroup$
    – Lord Shark the Unknown
    Dec 15 '18 at 10:30
















$begingroup$
The number of permutations of ${1,ldots,n+1}$ with $p(n+1)=n$ is simply the number of permutations of ${1,ldots,n}$; compose any such permutation with the transposition $(n n+1)$ yields a bijection between these sets of permutations.
$endgroup$
– Servaes
Dec 15 '18 at 9:42






$begingroup$
The number of permutations of ${1,ldots,n+1}$ with $p(n+1)=n$ is simply the number of permutations of ${1,ldots,n}$; compose any such permutation with the transposition $(n n+1)$ yields a bijection between these sets of permutations.
$endgroup$
– Servaes
Dec 15 '18 at 9:42














$begingroup$
But in our case, we count the permutations with $p(i)neq i$, are you sure there is such bijection? Because when $n+1$ takes $n$'s place, we dont have to worry about $p(n) neq n$ anymore.
$endgroup$
– Theorem
Dec 15 '18 at 9:46




$begingroup$
But in our case, we count the permutations with $p(i)neq i$, are you sure there is such bijection? Because when $n+1$ takes $n$'s place, we dont have to worry about $p(n) neq n$ anymore.
$endgroup$
– Theorem
Dec 15 '18 at 9:46












$begingroup$
Ah, so you mean to count the number of derangements with $p(n+1)neq n$? I only read your question at a glance, and commented on the last sentence.
$endgroup$
– Servaes
Dec 15 '18 at 9:49




$begingroup$
Ah, so you mean to count the number of derangements with $p(n+1)neq n$? I only read your question at a glance, and commented on the last sentence.
$endgroup$
– Servaes
Dec 15 '18 at 9:49












$begingroup$
Yes, I didn't say exactly derangements because that would imply $p(n+1)neq n+1$, but it is very similar.
$endgroup$
– Theorem
Dec 15 '18 at 9:50




$begingroup$
Yes, I didn't say exactly derangements because that would imply $p(n+1)neq n+1$, but it is very similar.
$endgroup$
– Theorem
Dec 15 '18 at 9:50












$begingroup$
In a derangement, $n+1$ is as likely to go to $n$ as to any given element of ${1,ldots,n}$.
$endgroup$
– Lord Shark the Unknown
Dec 15 '18 at 10:30




$begingroup$
In a derangement, $n+1$ is as likely to go to $n$ as to any given element of ${1,ldots,n}$.
$endgroup$
– Lord Shark the Unknown
Dec 15 '18 at 10:30










2 Answers
2






active

oldest

votes


















1












$begingroup$

Call a permutation $sigma$ of ${1,ldots,n+1}$ good if $sigma(i)neq i$ for all $1leq ileq n$ and and $sigma(n+1)neq n$.



Let $sigma$ be a good permutation, and let $a:=sigma(n+1)$ and $b:=sigma^{-1}(n+1)$, so that $aneq n$. If $aneq b$ then the permutation $(a n+1)sigma$ fixes $n+1$ and is a derangement of ${1,ldots,n}$. If $a=b$ then $(a n+1)sigma$ fixes $a$ and $n+1$, and is a derangement of ${1,ldots,n}setminus{a}$.



Conversely, for any derangement $sigma$ of ${1,ldots,n}$ and any $ain{1,ldots,n+1}$ with $aneq n$ the composition $(a n+1)sigma$ is good. Also, for any $ain{1,ldots,n-1}$ and any derangement $sigma$ of ${1,ldots,n}setminus{a}$ the composition $(a n+1)sigma$ is good.



This shows that the number of good permutations is $nD_n+(n-1)D_{n-1}$, where $D_m$ denotes the number of derangements of ${1,ldots,m}$.






share|cite|improve this answer











$endgroup$





















    1












    $begingroup$

    These kind of problems (permutations with forbidden positions) can be elegantly solved using rook numbers/polynomials:




    Let $P subseteq {1, ldots, n}^2$ be the diagram of forbidden positions. The number of permutations $p in S_n$ such that $(i,p(i)) notin P$ for all $i = 1, ldots, n$ is given by
    $$sum_{k=0}^n (-1)^k(n-k)!r_k$$
    where $r_k$ is the number of ways to place $k$ nonattacking rooks on the diagram $P$.




    In our case the diagram e.g. for $n=4$ (the board is then $5 times 5$) is given by:



    enter image description here



    $0$ rooks can be placed in one way on $P$. To place $k$ rooks on $P$, one can place a rook on one of the two positions in the last column of $P$ in $2$ ways and then place $k-1$ rooks on $n-1$ remaining positions, or one can just place all $k$ rooks in the first $n-1$ columns, ignoring the last one. Hence
    $$r_k = begin{cases}
    1, text{ if }k=0\
    2{n-1 choose k-1} + {n-1 choose k}, text{ if }k ge 1
    end{cases} = begin{cases}
    1, text{ if }k=0\
    {n choose k} + {n-1 choose k-1}, text{ if }k ge 1
    end{cases}$$



    Therefore the result is
    begin{align}
    sum_{k=0}^n (-1)^k(n-k)!r_k &= sum_{k=0}^n (-1)^k(n-k)!{n choose k} + n! + sum_{k=1}^n (-1)^k(n-k)!{n-1 choose k-1} \
    &= D_n + n! - sum_{j=0}^{n-1} (-1)^j((n-1)-j)!{n-1 choose j} \
    &= D_n + n! - D_{n-1}
    end{align}






    share|cite|improve this answer











    $endgroup$













    • $begingroup$
      Is there no simple solutions without pulling any big guns? The problem shouldn't be this complicated and I think I'm missing a key principle. (In addition I'm not sure the solution you achieved is correct)
      $endgroup$
      – Theorem
      Dec 15 '18 at 11:23











    Your Answer





    StackExchange.ifUsing("editor", function () {
    return StackExchange.using("mathjaxEditing", function () {
    StackExchange.MarkdownEditor.creationCallbacks.add(function (editor, postfix) {
    StackExchange.mathjaxEditing.prepareWmdForMathJax(editor, postfix, [["$", "$"], ["\\(","\\)"]]);
    });
    });
    }, "mathjax-editing");

    StackExchange.ready(function() {
    var channelOptions = {
    tags: "".split(" "),
    id: "69"
    };
    initTagRenderer("".split(" "), "".split(" "), channelOptions);

    StackExchange.using("externalEditor", function() {
    // Have to fire editor after snippets, if snippets enabled
    if (StackExchange.settings.snippets.snippetsEnabled) {
    StackExchange.using("snippets", function() {
    createEditor();
    });
    }
    else {
    createEditor();
    }
    });

    function createEditor() {
    StackExchange.prepareEditor({
    heartbeatType: 'answer',
    autoActivateHeartbeat: false,
    convertImagesToLinks: true,
    noModals: true,
    showLowRepImageUploadWarning: true,
    reputationToPostImages: 10,
    bindNavPrevention: true,
    postfix: "",
    imageUploader: {
    brandingHtml: "Powered by u003ca class="icon-imgur-white" href="https://imgur.com/"u003eu003c/au003e",
    contentPolicyHtml: "User contributions licensed under u003ca href="https://creativecommons.org/licenses/by-sa/3.0/"u003ecc by-sa 3.0 with attribution requiredu003c/au003e u003ca href="https://stackoverflow.com/legal/content-policy"u003e(content policy)u003c/au003e",
    allowUrls: true
    },
    noCode: true, onDemand: true,
    discardSelector: ".discard-answer"
    ,immediatelyShowMarkdownHelp:true
    });


    }
    });














    draft saved

    draft discarded


















    StackExchange.ready(
    function () {
    StackExchange.openid.initPostLogin('.new-post-login', 'https%3a%2f%2fmath.stackexchange.com%2fquestions%2f3040322%2fderangements-p-of-1-2-dots-n-n1-such-that-n1-doesnt-go-to-n%23new-answer', 'question_page');
    }
    );

    Post as a guest















    Required, but never shown

























    2 Answers
    2






    active

    oldest

    votes








    2 Answers
    2






    active

    oldest

    votes









    active

    oldest

    votes






    active

    oldest

    votes









    1












    $begingroup$

    Call a permutation $sigma$ of ${1,ldots,n+1}$ good if $sigma(i)neq i$ for all $1leq ileq n$ and and $sigma(n+1)neq n$.



    Let $sigma$ be a good permutation, and let $a:=sigma(n+1)$ and $b:=sigma^{-1}(n+1)$, so that $aneq n$. If $aneq b$ then the permutation $(a n+1)sigma$ fixes $n+1$ and is a derangement of ${1,ldots,n}$. If $a=b$ then $(a n+1)sigma$ fixes $a$ and $n+1$, and is a derangement of ${1,ldots,n}setminus{a}$.



    Conversely, for any derangement $sigma$ of ${1,ldots,n}$ and any $ain{1,ldots,n+1}$ with $aneq n$ the composition $(a n+1)sigma$ is good. Also, for any $ain{1,ldots,n-1}$ and any derangement $sigma$ of ${1,ldots,n}setminus{a}$ the composition $(a n+1)sigma$ is good.



    This shows that the number of good permutations is $nD_n+(n-1)D_{n-1}$, where $D_m$ denotes the number of derangements of ${1,ldots,m}$.






    share|cite|improve this answer











    $endgroup$


















      1












      $begingroup$

      Call a permutation $sigma$ of ${1,ldots,n+1}$ good if $sigma(i)neq i$ for all $1leq ileq n$ and and $sigma(n+1)neq n$.



      Let $sigma$ be a good permutation, and let $a:=sigma(n+1)$ and $b:=sigma^{-1}(n+1)$, so that $aneq n$. If $aneq b$ then the permutation $(a n+1)sigma$ fixes $n+1$ and is a derangement of ${1,ldots,n}$. If $a=b$ then $(a n+1)sigma$ fixes $a$ and $n+1$, and is a derangement of ${1,ldots,n}setminus{a}$.



      Conversely, for any derangement $sigma$ of ${1,ldots,n}$ and any $ain{1,ldots,n+1}$ with $aneq n$ the composition $(a n+1)sigma$ is good. Also, for any $ain{1,ldots,n-1}$ and any derangement $sigma$ of ${1,ldots,n}setminus{a}$ the composition $(a n+1)sigma$ is good.



      This shows that the number of good permutations is $nD_n+(n-1)D_{n-1}$, where $D_m$ denotes the number of derangements of ${1,ldots,m}$.






      share|cite|improve this answer











      $endgroup$
















        1












        1








        1





        $begingroup$

        Call a permutation $sigma$ of ${1,ldots,n+1}$ good if $sigma(i)neq i$ for all $1leq ileq n$ and and $sigma(n+1)neq n$.



        Let $sigma$ be a good permutation, and let $a:=sigma(n+1)$ and $b:=sigma^{-1}(n+1)$, so that $aneq n$. If $aneq b$ then the permutation $(a n+1)sigma$ fixes $n+1$ and is a derangement of ${1,ldots,n}$. If $a=b$ then $(a n+1)sigma$ fixes $a$ and $n+1$, and is a derangement of ${1,ldots,n}setminus{a}$.



        Conversely, for any derangement $sigma$ of ${1,ldots,n}$ and any $ain{1,ldots,n+1}$ with $aneq n$ the composition $(a n+1)sigma$ is good. Also, for any $ain{1,ldots,n-1}$ and any derangement $sigma$ of ${1,ldots,n}setminus{a}$ the composition $(a n+1)sigma$ is good.



        This shows that the number of good permutations is $nD_n+(n-1)D_{n-1}$, where $D_m$ denotes the number of derangements of ${1,ldots,m}$.






        share|cite|improve this answer











        $endgroup$



        Call a permutation $sigma$ of ${1,ldots,n+1}$ good if $sigma(i)neq i$ for all $1leq ileq n$ and and $sigma(n+1)neq n$.



        Let $sigma$ be a good permutation, and let $a:=sigma(n+1)$ and $b:=sigma^{-1}(n+1)$, so that $aneq n$. If $aneq b$ then the permutation $(a n+1)sigma$ fixes $n+1$ and is a derangement of ${1,ldots,n}$. If $a=b$ then $(a n+1)sigma$ fixes $a$ and $n+1$, and is a derangement of ${1,ldots,n}setminus{a}$.



        Conversely, for any derangement $sigma$ of ${1,ldots,n}$ and any $ain{1,ldots,n+1}$ with $aneq n$ the composition $(a n+1)sigma$ is good. Also, for any $ain{1,ldots,n-1}$ and any derangement $sigma$ of ${1,ldots,n}setminus{a}$ the composition $(a n+1)sigma$ is good.



        This shows that the number of good permutations is $nD_n+(n-1)D_{n-1}$, where $D_m$ denotes the number of derangements of ${1,ldots,m}$.







        share|cite|improve this answer














        share|cite|improve this answer



        share|cite|improve this answer








        edited Dec 15 '18 at 20:17

























        answered Dec 15 '18 at 19:57









        ServaesServaes

        27.5k34098




        27.5k34098























            1












            $begingroup$

            These kind of problems (permutations with forbidden positions) can be elegantly solved using rook numbers/polynomials:




            Let $P subseteq {1, ldots, n}^2$ be the diagram of forbidden positions. The number of permutations $p in S_n$ such that $(i,p(i)) notin P$ for all $i = 1, ldots, n$ is given by
            $$sum_{k=0}^n (-1)^k(n-k)!r_k$$
            where $r_k$ is the number of ways to place $k$ nonattacking rooks on the diagram $P$.




            In our case the diagram e.g. for $n=4$ (the board is then $5 times 5$) is given by:



            enter image description here



            $0$ rooks can be placed in one way on $P$. To place $k$ rooks on $P$, one can place a rook on one of the two positions in the last column of $P$ in $2$ ways and then place $k-1$ rooks on $n-1$ remaining positions, or one can just place all $k$ rooks in the first $n-1$ columns, ignoring the last one. Hence
            $$r_k = begin{cases}
            1, text{ if }k=0\
            2{n-1 choose k-1} + {n-1 choose k}, text{ if }k ge 1
            end{cases} = begin{cases}
            1, text{ if }k=0\
            {n choose k} + {n-1 choose k-1}, text{ if }k ge 1
            end{cases}$$



            Therefore the result is
            begin{align}
            sum_{k=0}^n (-1)^k(n-k)!r_k &= sum_{k=0}^n (-1)^k(n-k)!{n choose k} + n! + sum_{k=1}^n (-1)^k(n-k)!{n-1 choose k-1} \
            &= D_n + n! - sum_{j=0}^{n-1} (-1)^j((n-1)-j)!{n-1 choose j} \
            &= D_n + n! - D_{n-1}
            end{align}






            share|cite|improve this answer











            $endgroup$













            • $begingroup$
              Is there no simple solutions without pulling any big guns? The problem shouldn't be this complicated and I think I'm missing a key principle. (In addition I'm not sure the solution you achieved is correct)
              $endgroup$
              – Theorem
              Dec 15 '18 at 11:23
















            1












            $begingroup$

            These kind of problems (permutations with forbidden positions) can be elegantly solved using rook numbers/polynomials:




            Let $P subseteq {1, ldots, n}^2$ be the diagram of forbidden positions. The number of permutations $p in S_n$ such that $(i,p(i)) notin P$ for all $i = 1, ldots, n$ is given by
            $$sum_{k=0}^n (-1)^k(n-k)!r_k$$
            where $r_k$ is the number of ways to place $k$ nonattacking rooks on the diagram $P$.




            In our case the diagram e.g. for $n=4$ (the board is then $5 times 5$) is given by:



            enter image description here



            $0$ rooks can be placed in one way on $P$. To place $k$ rooks on $P$, one can place a rook on one of the two positions in the last column of $P$ in $2$ ways and then place $k-1$ rooks on $n-1$ remaining positions, or one can just place all $k$ rooks in the first $n-1$ columns, ignoring the last one. Hence
            $$r_k = begin{cases}
            1, text{ if }k=0\
            2{n-1 choose k-1} + {n-1 choose k}, text{ if }k ge 1
            end{cases} = begin{cases}
            1, text{ if }k=0\
            {n choose k} + {n-1 choose k-1}, text{ if }k ge 1
            end{cases}$$



            Therefore the result is
            begin{align}
            sum_{k=0}^n (-1)^k(n-k)!r_k &= sum_{k=0}^n (-1)^k(n-k)!{n choose k} + n! + sum_{k=1}^n (-1)^k(n-k)!{n-1 choose k-1} \
            &= D_n + n! - sum_{j=0}^{n-1} (-1)^j((n-1)-j)!{n-1 choose j} \
            &= D_n + n! - D_{n-1}
            end{align}






            share|cite|improve this answer











            $endgroup$













            • $begingroup$
              Is there no simple solutions without pulling any big guns? The problem shouldn't be this complicated and I think I'm missing a key principle. (In addition I'm not sure the solution you achieved is correct)
              $endgroup$
              – Theorem
              Dec 15 '18 at 11:23














            1












            1








            1





            $begingroup$

            These kind of problems (permutations with forbidden positions) can be elegantly solved using rook numbers/polynomials:




            Let $P subseteq {1, ldots, n}^2$ be the diagram of forbidden positions. The number of permutations $p in S_n$ such that $(i,p(i)) notin P$ for all $i = 1, ldots, n$ is given by
            $$sum_{k=0}^n (-1)^k(n-k)!r_k$$
            where $r_k$ is the number of ways to place $k$ nonattacking rooks on the diagram $P$.




            In our case the diagram e.g. for $n=4$ (the board is then $5 times 5$) is given by:



            enter image description here



            $0$ rooks can be placed in one way on $P$. To place $k$ rooks on $P$, one can place a rook on one of the two positions in the last column of $P$ in $2$ ways and then place $k-1$ rooks on $n-1$ remaining positions, or one can just place all $k$ rooks in the first $n-1$ columns, ignoring the last one. Hence
            $$r_k = begin{cases}
            1, text{ if }k=0\
            2{n-1 choose k-1} + {n-1 choose k}, text{ if }k ge 1
            end{cases} = begin{cases}
            1, text{ if }k=0\
            {n choose k} + {n-1 choose k-1}, text{ if }k ge 1
            end{cases}$$



            Therefore the result is
            begin{align}
            sum_{k=0}^n (-1)^k(n-k)!r_k &= sum_{k=0}^n (-1)^k(n-k)!{n choose k} + n! + sum_{k=1}^n (-1)^k(n-k)!{n-1 choose k-1} \
            &= D_n + n! - sum_{j=0}^{n-1} (-1)^j((n-1)-j)!{n-1 choose j} \
            &= D_n + n! - D_{n-1}
            end{align}






            share|cite|improve this answer











            $endgroup$



            These kind of problems (permutations with forbidden positions) can be elegantly solved using rook numbers/polynomials:




            Let $P subseteq {1, ldots, n}^2$ be the diagram of forbidden positions. The number of permutations $p in S_n$ such that $(i,p(i)) notin P$ for all $i = 1, ldots, n$ is given by
            $$sum_{k=0}^n (-1)^k(n-k)!r_k$$
            where $r_k$ is the number of ways to place $k$ nonattacking rooks on the diagram $P$.




            In our case the diagram e.g. for $n=4$ (the board is then $5 times 5$) is given by:



            enter image description here



            $0$ rooks can be placed in one way on $P$. To place $k$ rooks on $P$, one can place a rook on one of the two positions in the last column of $P$ in $2$ ways and then place $k-1$ rooks on $n-1$ remaining positions, or one can just place all $k$ rooks in the first $n-1$ columns, ignoring the last one. Hence
            $$r_k = begin{cases}
            1, text{ if }k=0\
            2{n-1 choose k-1} + {n-1 choose k}, text{ if }k ge 1
            end{cases} = begin{cases}
            1, text{ if }k=0\
            {n choose k} + {n-1 choose k-1}, text{ if }k ge 1
            end{cases}$$



            Therefore the result is
            begin{align}
            sum_{k=0}^n (-1)^k(n-k)!r_k &= sum_{k=0}^n (-1)^k(n-k)!{n choose k} + n! + sum_{k=1}^n (-1)^k(n-k)!{n-1 choose k-1} \
            &= D_n + n! - sum_{j=0}^{n-1} (-1)^j((n-1)-j)!{n-1 choose j} \
            &= D_n + n! - D_{n-1}
            end{align}







            share|cite|improve this answer














            share|cite|improve this answer



            share|cite|improve this answer








            edited Dec 15 '18 at 10:53

























            answered Dec 15 '18 at 10:47









            mechanodroidmechanodroid

            28.4k62548




            28.4k62548












            • $begingroup$
              Is there no simple solutions without pulling any big guns? The problem shouldn't be this complicated and I think I'm missing a key principle. (In addition I'm not sure the solution you achieved is correct)
              $endgroup$
              – Theorem
              Dec 15 '18 at 11:23


















            • $begingroup$
              Is there no simple solutions without pulling any big guns? The problem shouldn't be this complicated and I think I'm missing a key principle. (In addition I'm not sure the solution you achieved is correct)
              $endgroup$
              – Theorem
              Dec 15 '18 at 11:23
















            $begingroup$
            Is there no simple solutions without pulling any big guns? The problem shouldn't be this complicated and I think I'm missing a key principle. (In addition I'm not sure the solution you achieved is correct)
            $endgroup$
            – Theorem
            Dec 15 '18 at 11:23




            $begingroup$
            Is there no simple solutions without pulling any big guns? The problem shouldn't be this complicated and I think I'm missing a key principle. (In addition I'm not sure the solution you achieved is correct)
            $endgroup$
            – Theorem
            Dec 15 '18 at 11:23


















            draft saved

            draft discarded




















































            Thanks for contributing an answer to Mathematics Stack Exchange!


            • Please be sure to answer the question. Provide details and share your research!

            But avoid



            • Asking for help, clarification, or responding to other answers.

            • Making statements based on opinion; back them up with references or personal experience.


            Use MathJax to format equations. MathJax reference.


            To learn more, see our tips on writing great answers.




            draft saved


            draft discarded














            StackExchange.ready(
            function () {
            StackExchange.openid.initPostLogin('.new-post-login', 'https%3a%2f%2fmath.stackexchange.com%2fquestions%2f3040322%2fderangements-p-of-1-2-dots-n-n1-such-that-n1-doesnt-go-to-n%23new-answer', 'question_page');
            }
            );

            Post as a guest















            Required, but never shown





















































            Required, but never shown














            Required, but never shown












            Required, but never shown







            Required, but never shown

































            Required, but never shown














            Required, but never shown












            Required, but never shown







            Required, but never shown







            Popular posts from this blog

            Plaza Victoria

            In PowerPoint, is there a keyboard shortcut for bulleted / numbered list?

            How to put 3 figures in Latex with 2 figures side by side and 1 below these side by side images but in...